Triangle Inequality for integrals proof

Click For Summary
The discussion focuses on proving the inequality ∫|x+y| ≤ ∫|x| + ∫|y| using the triangle inequality. The user attempts to manipulate the integral by introducing a variable g, leading to the conclusion that ∫|x+y| can be bounded by the sum of the integrals of |x| and |y|. However, there is uncertainty about the validity of applying the triangle inequality in this context. The user acknowledges that the proof relies on the linearity of integration and the preservation of inequalities during integration. Overall, the conversation highlights the need for a clearer understanding of how these mathematical principles interact in the context of integrals.
Henry365
Messages
3
Reaction score
0

Homework Statement


What I want to show is this:
∫|x+y| ≤ ∫|x| + ∫|y|

Homework Equations


|x+y| ≤ |x| + |y|


The Attempt at a Solution



So I thought if I used the triangle inequality I could get to something along the lines of:

Lets g belong to the real numbers
∫|x+y| = ∫|x+g-g+y|≤ ∫|x+g| + |y-g|= ∫|x+g| + ∫|y-g|

As g belongs to the reals it can be zero meaning ∫|x+y| ≤ ∫|x| + ∫|y|.

Now the problem with this is that is uses the triangle inequality and I have no idea if the triangle inequality works this way, and if it does I need to prove it, and I have no idea about where to start that from.
 
Physics news on Phys.org
I suppose I am necro-posting here, but this result follows from two facts. One is the linearity of integration:
\int (f(x)+g(x)) \, dx = \int f(x) \, dx + \int g(x) \, dx,
and the other is that integration preserves inequalities: if f(x) \le g(x) on the interval [a,b], then
\int_{a}^{b}f(x) \, dx \le \int_{a}^{b} g(x) \, dx.
 
Question: A clock's minute hand has length 4 and its hour hand has length 3. What is the distance between the tips at the moment when it is increasing most rapidly?(Putnam Exam Question) Answer: Making assumption that both the hands moves at constant angular velocities, the answer is ## \sqrt{7} .## But don't you think this assumption is somewhat doubtful and wrong?

Similar threads

Replies
9
Views
1K
Replies
1
Views
1K
  • · Replies 9 ·
Replies
9
Views
2K
Replies
3
Views
2K
  • · Replies 9 ·
Replies
9
Views
2K
  • · Replies 3 ·
Replies
3
Views
2K
  • · Replies 14 ·
Replies
14
Views
2K
  • · Replies 6 ·
Replies
6
Views
2K
Replies
10
Views
2K
  • · Replies 3 ·
Replies
3
Views
1K